In a finite group G, the inverse of each element is a power of itself.

Click For Summary
In a finite group G, each element g has a finite order, meaning there exists a positive integer r such that g^r equals the identity element e. The Lagrange theorem indicates that the order of any element divides the order of the group, ensuring that g^n = e for some n. A proof by contradiction shows that if no such k exists for g, it leads to an infinite set of distinct powers of g, which contradicts the finiteness of G. Therefore, the assumption must be false, confirming that the inverse of each element is indeed a power of itself. This conclusion relies on established group properties, including the laws of exponents.
gotmilk04
Messages
44
Reaction score
0

Homework Statement


If G is a finite group and g is in G, then there exists a positive integer r with g^r=e.
and in general, prove that, in any finite group G, the inverse of each element is a power of itself.


Homework Equations





The Attempt at a Solution


I know if a group is finite then it has a finite order, which means g^n=e, but how do you write that as a proof?
 
Physics news on Phys.org
Be careful: that the order of the group is finite does not automatically mean that g^n = e for some n.

Did you learn about the Lagrange theorem yet?
It says that the order of any element of the group, divides the order of the group. When you apply that you know that in particular, g^n = e for some finite number n and you can write down an explicit inverse for g (which, as you probably know, is unique).
 
What about:

Assume on the contrary that there does not exist any k \in \mathbb{Z}^{+} such that, for g \in G, g^{k} = 1_{G}, the identity. As G is finite, we can't have all g^{k} distinct for all positive integers k, as it would result in a set having infinite cardinality. Thus \exists k_{1}, k_{2} \in \mathbb{Z}_{+} such that k_{1} \neq k_{2}, g^{k_{1}} = g^{k_{2}}. Without loss of generality we may assume that k_{1} > k_{2}. But then g^{k_{1}}g^{-k_{2}} = g^{k_{2}}g^{-k{2}} \Leftrightarrow g^{k_{1}-k_{2}} = 1_{G}, and g_{1}-g_{2} \in \mathbb{Z}^{+}, which is a contradiction. Thus the assumtion that such an integer k exists must be false, and the assertion is proven.

This is actually pretty much what you stated, but formulated in such a way that it provides a proof. Though, one must be careful. For example, this proof uses the "laws of exponents" for groups, which has to be proven. Though, I suspect you have already done that in your course.
 
Question: A clock's minute hand has length 4 and its hour hand has length 3. What is the distance between the tips at the moment when it is increasing most rapidly?(Putnam Exam Question) Answer: Making assumption that both the hands moves at constant angular velocities, the answer is ## \sqrt{7} .## But don't you think this assumption is somewhat doubtful and wrong?

Similar threads

Replies
3
Views
1K
  • · Replies 3 ·
Replies
3
Views
2K
Replies
11
Views
2K
  • · Replies 2 ·
Replies
2
Views
2K
  • · Replies 1 ·
Replies
1
Views
2K
Replies
1
Views
2K
  • · Replies 2 ·
Replies
2
Views
4K
  • · Replies 1 ·
Replies
1
Views
5K
  • · Replies 3 ·
Replies
3
Views
2K
  • · Replies 3 ·
Replies
3
Views
2K